Unendliches Widerstandsnetzwerk

"moritz erbs_löh"

Rh+Rh+Rv für das 2. Glied?

Ich hab das Problem nur aufgeteilt. In serielle und parallele Widerstände.

nicht auch unterschiedlich sein?

Nein, das ist nur eine Formel. eine rekursive. Jetzt muss man den nur in einen nichtrekursiven umwandeln. Wie das ging weiß ich abe rnicht mehr.

Reply to
Markus
Loading thread data ...

Das ist das schöne an dieser Lösung. Man baucht keinen Grenzwert berechnen.

Gar nicht. Es handelt sich um den fertigen Grenzwert. Endliche Strukturen musst du als Kettenbruch darstellen:

Rh + 1/(Gv+1/(Rh+1/(Gv+1/ .... das ganze ist relativ simpel. Der geschlossene Ausdruck für soetwas ist eine spezielle Variante der Determinanten (Konvergenten und Kontinuanten). Lies dich ein, ist einfacher, als du denkst.

Bitte schön:

formatting link

Die nte Konvergente ist, was du suchst.

-- lg Johannes

Reply to
John F

Lies nach :-)

formatting link

lg Johannes

Reply to
John F

"Henning Paul" schrieb im Newsbeitrag news: snipped-for-privacy@fump.de.vu...

formatting link

Genau sowas habe ich gesucht, ausreichend detailiert beschrieben. Ich hätte also nach Hochfrequenz suchen sollen-

Im enddefekt will ich auch darauf hinaus.

R' und G' sind dann "Belage" oder? Kann ich sie nicht durch Belage der Art Rh/Länge ersetzen? Wie würde ich sonst zu R' und G' gelangen?

Mfg Moritz

Reply to
moritz erbs_löh

moritz erbs_löh schrieb:

Ich dachte du wolltest noch eine Formel die für beliebige n's verwendbar ist, also wo n enthalten sein muss. Ansonsten wird das schon stimmen, es ist ja leicht einzusehen dass der Effekt eines zusätzlichen Gliedes immer kleiner wird, je mehr man schon hat. Bei theoretisch unendlich vielen Gliedern gibt es also keinen Unterschied mehr, egal ob man das Glied vorne oder hinten hinzufügt. Bin gerade am überlegen wie man beim Wellenwiderstand vorgehen muss, um Rw = sqrt(L/C) zu erhalten...

Gruß Jens

Reply to
Jens Dierks

ich habe interessehalber mal den Vierpolansatz gewählt und komme auf das gleiche Ergebnis, also wird das schon stimmen. Man muß dann allerdings die Vierpolkoeffizienten bestimmen, was mehr Aufwand ist als der Ansatz von Michael.

Auf den ersten Blick kann man da einen unendlichen Kettenbruch sehen, nur wüßte ich nicht, wie man damit eine Grenzwertbetrachtung einer Reihe oder gar eines Integrals machen sollte. Der Ansatz von Michael ist eigentlich unmittelbar einsichtig und man braucht da IMHO nichts mehr zu beweisen, denke mal das wird die einfachste Lösung sein.

Du kannst die Kette auch abbrechen lassen und mit dem Eingangswiderstand der unendlichen Kette abschließen, genauso wie man es mit dem Wellenwiderstand bei Vierpolen macht. Aber auf jeden Fall hat man mal wieder was dazugelernt.

mfg. Winfried

Reply to
Winfried Salomon

"moritz erbs_löh" schrieb im Newsbeitrag news:44522926$0$21836$ snipped-for-privacy@read.cnntp.org...

und

hin,

und

Hallo Moritz,

ich habe von rechts mit der Berechnung angefangen.

Rin_1 = Rh + Rv

Rin_2 = Rh + 1/(1/Rv+1/Rin_1)

Rin_3 = Rh + 1/(1/Rv+1/Rin_2)

Rin_4 = Rh + 1/(1/Rv+1/Rin_3) ...

Genaugenommen ist das ein Kettebruch mit n "Ebenen". Allerdings ist es wohl ziemlich sinnlos den z.B, für 10 Glieder auszumultiplizieren. Das läßt sich viel einfacher in eine for-Schleife packen. Siehe das angehängte Perl-Programm.

# 1 Glied $Rin = $Rh + $Rv ;

# 2 bis n Glieder for ($n = 2; $n Dabei hat das Problem Ahnlichkeit mit der Berechnung des Wellenwiderstandes

eine

Also ohne L und C ist jede Rechnung in Richtung Wellenwiderstand Unsinn! Ich denke da mit Grausen an eines der Postings mit dem Grenzübergang R=Wurzel(Rh/Rv) Wo bitte bleibt da die Einheit Ohm!

Gruß Helmut

#!/usr/bin/perl # Einganswiderstand eines endlichen R-Leiternetzwerks # # --Rh--+--Rh--+--Rh--+-- # | | | # Rin --> Rv Rv Rv # | | | # ------+------+------+--

# Beispiel $Rh = 1 ; $Rv = 2 ;

# Anzahl der Glieder $nmax = 8 ;

print "\nEingangswiderstand eines R-Leiternetzwerks mit n-Gliedern\n\n" ;

print " --Rh--+--Rh--+--Rh--+--\n" ; print " | | | \n" ; print " Rn --> Rv Rv Rv \n" ; print " | | | \n" ; print " ------+------+------+--\n\n" ;

print " Rh = $Rh, Rv = $Rv, 1 bis $nmax Glider\n\n" ;

# 1 Glied $Rin = $Rh + $Rv ; print " R-1 = $Rin Ohm\n" ;

# 2 bis n Glieder for ($n = 2; $n Rv Rv Rv # | | | # ------+------+------+-- # # Rh = 1, Rv = 2, 1 bis 8 Glider

# R-1 = 3 Ohm # R-2 = 2.2 Ohm # R-3 = 2.04761904761905 Ohm # R-4 = 2.01176470588235 Ohm # R-5 = 2.00293255131965 Ohm # R-6 = 2.0007326007326 Ohm # R-7 = 2.0001831166453 Ohm # R-8 = 2.00004577706569 Ohm

Reply to
Helmut Sennewald

so

nde. Du brauchst wohl noch mehr Punkte von Dieter. Betrachte bitte folgende Netzwerke:

| | --- +--+--+ | | | | |2| --- --- | | | | | | --- |2| |2| | | | | | +--+--+ --- --- | | | |

--- --- --- --- | | | | | | | | |2| |2| |2| |2| | | | | | | | |

--- --- --- --- | | | | +--+--+ +--+--+ | |

Hier habe ich das Problem auch nur "aufgeteilt", aber das Linke Netz hat den Widerstand 3, das linke 2 [zu Ohm proportionale Einheit ihres Vertrauens bitte hier einf=FCgen].

Michael Karcher

Reply to
Michael Karcher

Ich doch extra dazugeschrieben dass das wahrscheinlich falsch ist. Man muss halt von hinten drangehen.

Reply to
Markus

"Helmut Sennewald" schrieb im Newsbeitrag news:e301bp$7th$01$ snipped-for-privacy@news.t-online.com...

Widerstandsnetzwerkes

dabei

Hallo an alle,

da muss ich mich glatt entschuldigen. Da stand ja R=Wurzel(R/G) und das ergibt im Ergebnis Ohm als Einheit.

Damit nehme ich meine obige Aussage zum Thema Wellenwiderstand sofort zurück.

Gruß Helmut

Reply to
Helmut Sennewald

"Helmut Sennewald" schrieb im Newsbeitrag news:e301bp$7th$01$ snipped-for-privacy@news.t-online.com...

Widerstandsnetzwerkes

dabei

Sorry,

da war ich zu schnell beim Lesen. Da stand ja R=Wurzel(R'/G') . Da kommt als Einheit doch Ohm heraus. Ich nehme alles obig gesagte bezüglich Wellenwiderstand zurück.

Im Grenzübergang für unendlich viele Glieder ergibt sich für obige Schaltung tatsächlich der Wert aus der Formel für den Wellenwiderstand.

Rin = Wurzel(Rh*Rv)

falls Rh

Reply to
Helmut Sennewald

"moritz erbs_löh" schrieb im Newsbeitrag

Um es etwas zu präzisieren Das diskrete unendliche Widerstandsnetzwerk sollte nur ein Ansatz sein, um später infinitesimale Rv und Rh-Beläge zu verwenden, denn das ganze soll schließlich für Wärmewiderstände gelten, wobei die Rv den Wärmewiderstand des Wärmeleiters darstellt und Rh den Wärmeübergangswiederstand zur Luft. Nun habe ich das ganze also einmal mit der Wellenwiderstands-Formel berechnet, und einmal mit der Formel für das unendliche Widerstandsnetzwerk, wobei ich die infinitesimalen dRv(dl) und dRh(dl) verwendet habe, und dann den Grenzwert für dl -> 0 genommen habe. Das tolle ist das bei beiden Ansätzen genau das selbe rauskam, nämlich:

Rth *b = 3,54 Wm/K, wobei b die Breite einer 35um Kupferlage ist. Für 1cm ergibt sich z.B. 354W/K, was allerdings etwas hoch scheint... Da bei beiden Ansätzen aber das selbe rauskam sollte das wohl stimmen.

Reply to
moritz erbs_löh

Michael Karcher schrieb:

[....]

Klappt nicht, um meinen Haaren Zeit zur Regeneration zu geben hab ich ihn z.Zt. im Filter.:-)

Gruß Dieter

Reply to
Dieter Wiedmann

Dieter Wiedmann schrieb:

Du hast noch Haare auf dem "Gletscher"?

--
gruß hdw
Reply to
horst-d.winzler

"horst-d.winzler" schrieb:

Ja, und auch Zähne hab ich noch, trotz vorhandener Tischkante!

Gruß Dieter

Reply to
Dieter Wiedmann

Dieter Wiedmann schrieb:

Ja, die Natur hats so eingerichtet, daß alles was nicht hinreichend benutzt wird, sich zurückbildet. Du benutzt doch sicher auswechselbare Tischkannten? ;-)

--
gruß hdw
Reply to
horst-d.winzler

Korrektur: Nicht alle Funktionen f lassen das zu. Wenn das Ergebnis von der Wahl der x_i unabhängig ist, kann das ja gerade keine Eigenschaft der Folge von Folgen ((x_i)_n) sein. (Nebenbei glaube ich nicht, dass Du wirklich dx_i in der Summe stehen haben willst, das sind in der üblichen Notation Elemente eines Tangentialraums. Besser: f(z_i)*(x_i-x_{i-1}).)

Da nicht einmal jede Reihe konvergent ist, ist das offensichtlich falsch. Zu jeder konvergenten Reihe ist die Aussage trivial, man nehme f(x)=r für 0

Reply to
Christopher Creutzig

ElectronDepot website is not affiliated with any of the manufacturers or service providers discussed here. All logos and trade names are the property of their respective owners.